Simplify -(3xy^4)^-4

Answers

Answer 1

Answer:

[tex]\frac{-1}{81x^4y^16}[/tex]

Step-by-step explanation:

→Rewrite the expression so that the exponent is no longer negative. To do this, you write it as:

[tex]\frac{-1}{(3xy^4)^4}[/tex]

→Apply the exponent (4) to the 3, x, and [tex]y^4[/tex]:

[tex]\frac{-1}{81x^4y^16}[/tex]


Related Questions

SOMEONE HELP ME PLEASE BECAUSE I NEED IT ASAP!!!!!!!

Some doctors recommend that no more than 30% of a person's daily calories come from fats. Following this recommendation, if you eat 2,400 calories in a day, what is the maximum number of calories that should come from fats?

Show your work below: Go back to this question
A:750
B:1,680
C:2,370
D:720

Answers

Answer:

D 720

Step-by-step explanation:

Take the max calories and multiply by the percent

2400*30%

Change to decimal form

2400 *.30

720

pls help for brainliest answer

Answers

The amount of yellow cubes is =x
Blue is 3x
Green is 15x
x+3x+15x=19x which is the total amount of cubes
Therefore the probability of picking a yellow cube is 1/19

What is the greatest common factor of 8m, 36m3, and 12?
A.4
B.8
C.4m
D.8m

Answers

Answer:

A

Step-by-step explanation:

The greatest common factor of 8m, 36m³, and 12 is 4.

4(2m), 4(9m³), and 4(3).

The HCF is the largest value that divides all three of them.

The greatest common factor of 8m, 36m³, and 12 is; 4.

How to find Greatest common factor?

Greatest common factor, as the name implies, is greatest among all common factors among the quantities given.

We could factorize the quantities in smallest relative factors possible (the level of factors over which all quantities can form themselves), and collect as many common factors as we could.

Factorize the given numbers and identify all common factors. To get the GCF(Greatest common factor) multiply all common factors;

The Common factors are: 2 and 2, their product is 2 x 2=4, then

4(2m) =  2 x 2 x2 x m

4(9m³) =  2 x 2 x3 x3 x m x m x m

4(3) = 2 x 2 x3

Hence, the greatest common factor is 4.

Learn more about greatest common factor here:

https://brainly.com/question/207329

#SPJ2

Please help, On Saturday morning, Owen earned $21. By the end of the afternoon he had earned a total of $60. Enter an equation, using x as your variable, to determine whether Owen earned $39 or $37 on Saturday afternoon.

Answers

Answer:

$39 is the correct answer .

Step-by-step explanation:

Let the amount he earned on the afternoon  be x then

Amount earned in morning + Amount earned in afternoon=total amount

[tex]\$ 21 +\ x\ =\ \$ 60 \\\ x=\$60 -\$ 21\\x=\$ 39[/tex]

Therefore $39 is the amount he earned on afternoon .

PLEASE HELP ... how many students are in each van? And how many students are in each bus ?

Answers

Answer:

7 students in each van and 52 students in each bus.

Step-by-step explanation:

v = no of students in van, b = no of student in bus

High school A ⇒ 3v + 6b = 333 ---------- Equation (1)

High school B ⇒ 11v + 6b = 389 ------------ Equation (2)

Solve equation (1) & (2) simultaneously :

11v + 6b = 389

3v + 6b = 333

----------------------- MINUS

8v + 0 = 56

v = 56 / 8

v = 7

Put v = 7 into Equation (1) to get b:

3v + 6b = 333

3 (7) + 6b = 333

21 + 6b = 333

6b = 312

b = 312 / 6

b = 52

So there are 7 students in each van and 52 students in each bus.

Scores of an IQ test have a​ bell-shaped distribution with a mean of 100 and a standard deviation of 20. Use the empirical rule to determine the followinWhat percentage of people has an IQ score greater than 120?

Answers

Answer:

Percentage of people with IQ greater than 120 is 84.134%

Step-by-step explanation:

First thing we do here is to calculate the

z-score

Mathematically;

z-score = (x-mean)/SD

here, x = 120, mean = 100 while SD = standard deviation = 20

z-score = (120-100)/20 = 20/20 = 1

The probability we want to calculate is;

P(x > 120) or P(z >1)

we proceed to use the standard score table to get this;

From the standard score table ;

P(z >1) = 0.84134

which is in percentage = 84.134%

what is = to 95
if u do iwill give u brainliest

Answers

Answer:

19 x 5

Step-by-step explanation:

[tex]19 \times 5 = 95[/tex]

19 X 5 that’s is it

what is 9.45 rounded to the nearest whole number?

Answers

Answer:

[tex]\Huge \boxed{\mathrm{9}}[/tex]

[tex]\rule[225]{225}{2}[/tex]

Step-by-step explanation:

9.45

To round up to nearest whole number, we have to see the tenth place.

If the value of the tenth place is greater or equal to 5, then we round up the number to 10. If the value of the tenth place is less than 5, the we round up the number to 9.

The value of the tenth place is 4, which is less than 5, we will round up the number to 9.

[tex]\rule[225]{225}{2}[/tex]

9.45 rounded to the nearest whole number is 9.

Given that a decimal number 9.45 we need to round it to the nearest whole number,

To round a decimal number to the nearest whole number, you need to look at the digit immediately to the right of the decimal point.

In the case of 9.45, the digit to the right of the decimal point is 4. If this digit is less than 5, you simply remove all the digits after the decimal point and keep the whole number as it is.

However, if the digit to the right of the decimal point is 5 or greater, you increase the whole number by 1.

To round 9.45 to the nearest whole number, we examine the decimal part of the number, which is 0.45.

If the decimal part is less than 0.5, we round down to the nearest whole number. In this case, since 0.45 is less than 0.5, we round down.

Therefore, 9.45 rounded to the nearest whole number is 9.

Learn more about Rounding numbers click;

https://brainly.com/question/29261078

#SPJ6


Simplify (13n2+11n−2n4)+(−13n2−3n−6n4) . Write the answer in standard form

Answers

Answer:

NaN ×10°

Step-by-step explanation:

Steps:

Let's simplify step-by-step.

13n2+11n−2n4−13n2−3n−6n4

=13n2+11n+−2n4+−13n2+−3n+−6n4

Combine Like Terms:

=13n2+11n+−2n4+−13n2+−3n+−6n4

=(−2n4+−6n4)+(13n2+−13n2)+(11n+−3n)

=−8n4+8n

Answer: =−8n4+8n

Convert to standard form: NaN ×10°

Hope this helps.

a Simplified form of  expression (13n²+11n−2n⁴)+(−13n²−3n−6n⁴) in the standard notation is n(2³ - n³).

What is simplification?

Simplifying procedures is one way to achieve uniformity in job efforts, expenses, and time. It reduces diversity and variation that is pointless, harmful, or unneeded. Parenthesis, exponents, multiplication, division, addition, and subtraction are all referred to as PEMDAS. The order of the letters in PEMDAS informs you what to calculate first, second, third, and so on, until the computation is finished, given two or more operations in a single statement.

Given an expression (13n²+11n−2n⁴)+(−13n²−3n−6n⁴)

Let's simplify it step-by-step.

13n²+11n−2n⁴−13n²−3n−6n⁴

=> 13n²+11n−2n⁴−13n²−3n−6n⁴

Combine Like Terms:

=> 13n²+11n−2n⁴−13n²−3n−6n⁴

=> 8n - 8n⁴

=>n(2³ - n³)

Therefore, a Simplified form of  expression (13n²+11n−2n⁴)+(−13n²−3n−6n⁴) in the standard notation is n(2³ - n³).

Learn more about Simplification here:

https://brainly.com/question/28996879

#SPJ2

Terry bought some gum and some candy. The number of packs of chewing gum was one more than the number of mints. The number of mints was three times the number of chocolate bars. If gum cost 6 cents a pack, mints cost 3 cents each, and chocolate bars cost 10 cents each, how many of each confection did he get for 80 cents?

Answers

Answer:

Hi!!

Step-by-step explanation:

7 = chewing gum

6 = mints

2 = chocolate

Answer:

number of packs of chewing gum is 7

number of mints  is 6

number of chocolate bars is 2

Step-by-step explanation:

number of packs of chewing gum → x,  cost 6¢

number of mints → y = x-1 =3z,  cost  3¢

number of chocolate bars → z = y/3 = (x-1)/3, cost 10¢

put in equation 6x + 3y +10z = 80 plug in term of x

6x +3(x-1) +10((x-1)/3) =80 multiply each side by 3

18x +9(x-1) +10(x-1) =240 distribute

18x+9x-9+10x-10 =240 combine like terms

37x-19 = 240 add 19 in each side

37x =259

x = 7

plug into y = x-1 = 7-1 = 6 so y=6

and  z = y/3 =6/3 =2 so z = 2

PLEASE HELP ASAP 50 POINTS WILL MARK BRAINLY A basketball is thrown upwards. The height f(t), in feet, of the basketball at time t, in seconds, is given by the following function: f(t) = −16t^2 + 44t + 12 Which of the following is a reasonable domain of the graph of the function when the basketball falls from its maximum height to the ground?

Answers

Answer:

Domain is 1.375 s to 3 s

Step-by-step explanation:

It is given that, A basketball is thrown upwards. The height f(t), in feet, of the basketball at time t, in seconds, is given by the following function:

f(t)= -16t²+44t+12

We have to find the domain of the graph of the function when the basketball falls from its maximum height to the ground. It is shown in the attached graph.

It is clear that, the domain of the graph is from 1.375 s to 3 s. Its maximum height is 42.25 feet.

please mark brainliest :)

Answer:

1,375

Step-by-step explanation:

Xv = -b/2a

Xv = -44/2*(-16)

Xv = 44/32

Xv = 1,375

Work out the volume of this prism.

Answers

Answer:

840

Step-by-step explanation:

Lets split the shape in 2 shapes which gives us 2 rectangular prisms

1st shape is the one stcking upward: 12*5*6=360

2nd shape: (we must subtract 5 from 15 because we used that for the other shape and hat wouldn't give us the correct answer) 10*8*6=480

480+360=

840

Need help ASAP please and thank you

Answers

Answer:

Step-by-step explanation:

The only angle we are given directly inside that triangle is angle R, 87 degrees. We can use the fact that the exterior angle at T is 92, so the interior angle is 180 - 92 = 88.  Now we can find angle S: 180 - 87 - 88 = 5. So angle S = 5 degrees. The Law of Sines can be used now to find the length of SR:

[tex]\frac{sin5}{950} =\frac{sin88}{x}[/tex]

Cross multiply to get

x sin5 = 950 sin88 and

[tex]x=\frac{950sin88}{sin5}[/tex] so

x = 10,893 km

Company A charges $17, plus $11 per day to rent a piece of equipment. Company B charges $33, plus $9 per day to rent the same piece of equipment. How many days must the piece of equipment be rented for Company B to be less expensive?

Answers

Answer: The answer is 8 days

Step-by-step explanation:

this is the last step you do

16/2    2/2

and 16/2

is 8

so the answer is 8

Hope this helps :)

A bank is offering a simple interest rate of 5% — that is, the bank will pay a fixed 5% of an initial investment as interest each year. By contrast, a stockbroker is offering a 4% interest rate compounded annually: 4% of the total value of the investment at the end of the year. If $1000 is invested in the bank and $1000 is invested with the stockbroker, after 4 years, what will be the total value of the two investments combined? Round to the nearest dollar.

Answers

25 would be the nearest to your question and complete 100 with the 4

Brainliest question please please help me please

Answers

S = 1/2 h * (a+b)
SA = sqroot of (4^2 + 1^2) =sqrt17
MK = sqrt of (11^2 + 3^2) = sqrt130
ST = sqrt of (4^2 + 1^2) = sqrt17
S = 1/2 * sqrt17 (sqrt17+sqrt30)=32 unit square

Solve -2/3x>8 or -2/3x<4

Answers

Answer:

u = -32

Step-by-step expla

Simplify

             

Answer:

(x | x< -12 or x> -6)

Step-by-step explanation:

-2/3x>8 ⇒ -2/3x*3/2>8*3/2 ⇒ -x> 12 ⇒ x< -12

-2/3x<4 ⇒ -2/3x*3/2<4*3/2 ⇒ -x< 6 ⇒ x> -6

(x | x< -12 or x> -6)

What is the perimeter of triangle ABC. Coordinates A (4,-1) B (-1,4) C (0,-3)

Answers

Answer:

18.61

Step-by-step explanation:

The length of one leg of a right triangle is 7 cm more than that of the other leg. The length of the hypotenuse is 3 cm more than double that of the shorter leg. Find the lengthzs of each of the three sides.

Answers

Answer:

The lengths are 5, 12, and 13.

Step-by-step explanation:

Let x represent the shorter leg of the triangle. Since the other leg is 7 cm more, the longer leg is x+7. Since the length of the hypotenuse is 3 cm more than double that of the shorter leg, the hypotenuse is 2x+3.

The Pythagorean Theorem may be used to find the lengths.

[tex]a^{2}[/tex] + [tex]b^{2}[/tex] = [tex]c^{2}[/tex]  a, b are the short and long lengths and c is the hypotenuse

→ [tex]x^{2} + (x+7)^{2} = (2x+3)^{2}[/tex]

→ [tex]x^{2} + x^{2} + 14x + 49 = 4x^{2} + 12x + 9[/tex]

→ [tex]2x^{2} + 14x + 49 = 4x^{2} + 12x + 9[/tex]

→ [tex]2x^{2} - 2x - 40 = 0[/tex]

→ [tex]x^{2} - x - 20 = 0[/tex]

→ [tex](x-5)(x+4) = 0[/tex]

→ [tex]x = 5[/tex] (The length cannot be negative)

The shorter leg, x, is 5 cm. Since the longer leg is x+7, it is 12 cm. Since the length of the hypotenuse is 2x+3, the hypotenuse is 13 cm.

x+3.8=5.73
what does x equal

Answers

Answer:

1.93

Step-by-step explanation:

5.71 - 3.8 = 1.93

Answer:

1.93

Step-by-step explanation:

100*9-2+12/2=

Just need the answer

Answers

Answer:

904

Step-by-step explanation:

According to PEMDAS, we must multiply 100 and 9 first, which gets us 900. Then divide 12 by 2, which gets us six. 900 - 2 + 6 = 898 + 6 = 904.

Answer:

904

Step-by-step explanation:

100 * 9 = 900 -2 =898 +12/2 →898+6= 904

12/2 = 6

Have a good day and stay safe!

Which situation involves a change modeled by a positive number but an end result modeled by a negative number? A.Leo enters an elevator on the second floor and goes down four floors. B. A plane, flying at an altitude of 300 feet, descends 50 feet. C. Irene is scuba diving at –30 feet. She swims 10 feet toward the surface to watch a school of fish. D. April owes her brother $5. On Friday, she gets $10 for her allowance.

Answers

The positive number but a negative result would be :

C. Irene is scuba diving at –30 feet. She swims 10 feet toward the surface to watch a school of fish.

The positive value is she swam 10 feet up to the surface, but she is still below the surface, which would be a negative result.

The situation that would involve a change that is from a positive number but ended in a negative number is option C.

This option tells us that at the negative number she was 30 feet below the surface of the water, while at positive number she was 10 feet up the water surface.

What is a positive number?

This is a number that is greater than 0.

What is a negative number?

This is a number that is less than 0.

Read more on positive and negative numbers here:

https://brainly.com/question/13831313

Hear is a rectangle 7 cm 3 cm the squre has the same peremeter as the rectangle what is the side length of this squre ______7cm_______ l l l l 3 cm l _______________l

Answers

Answer:

5 cm

Step-by-step explanation:

The rectangle has a length (l) of 7 cm and a width (w) of 3 cm. Therefore, the perimeter of the rectangle is:

Perimeter of rectangle = 2(l + w) = 2(7 + 3) = 2(10) = 20 cm

Since the rectangle and the square has the same perimeter, Therefore, the perimeter of the square is:

Perimeter of square = 2(l + w). But the length and width of a square are the same (i.e l = w). Therefore:

Perimeter of square = 4l

20 = 4l

l = 20 / 4 = 5 cm

Each side of the square has a length of 5 cm

Toby, Daoud and Elsa share a sum of money. Toby gets 1/6 of the money and Daoud gets 1/2 of the money. Elsa gets the rest of the money Toby gets £1.50 Work out how much money Daoud gets. Find the ratio amount of money Toby gets: amount of money Elsa gets Give your answer in the form a : b where a and are whole numbers

Answers

Answer:

Daoud's share of the money  = £4.50Toby's Money : Elsa's Money=1:2

Step-by-step explanation:

Let the total amount of the money = x

Toby gets 1/6 of the moneyToby gets £1.50

Therefore:

[tex]\dfrac16x=1.50\\x=1.50 \times 6\\x=\£$ 9[/tex]

Daoud gets 1/2 of the money.

Therefore: Daoud's share of the money

[tex]=\dfrac12 \times \£$ 9 =\£$ 4.50[/tex]

Elsa's Share of the money = 9-(4.50+1.50)=£3

Therefore:

Amount of money Toby gets: Amount of money Elsa gets

= 1.50: 3

Divide both sides by 1.50

Toby's Money : Elsa's Money=1:2

Answer:

Step-by-step explanation:

Help me please ASAP​

Answers

Answer:

135°

Step-by-step explanation:

total = 360°

360° = 90° + 135° + x°

x° = 360° - (90° + 135°) = 360° - 225°

x° = 135°

Which polynomial is prime? A. x^4 + 3x^2 – x^2 – 3 B. x^4 – 3x^2 – x^2 + 3 C. 3x^2 + x – 6x – 2 D. 3x^2 + x – 6x + 3

Answers

Answer:

A and B

Step-by-step explanation:

A polynomial with integer coefficients that cannot be factored into polynomials of lower degree, also with integer coefficients, is called an irreducible or prime polynomial  

As  per above definition, A and B polynomials have coefficients 1 or 3, so are prime polynomials.

C and D polynomials have coefficient of 6 which can be further factored

A. x^4 + 3x^2 – x^2 – 3

yes

B. x^4 – 3x^2 – x^2 + 3

yes

C. 3x^2 + x – 6x – 2

no

D. 3x^2 + x – 6x + 3

no

six times the sum of nine and a number

Answers

Answer:

Brainliest plz

Step-by-step explanation:

let use x as 'a number'

6 [tex]*[/tex] (9+x)

or

6 * (x+9)

Six times the sum of nine and the number x is 54+6x.

Suppose the number is x.

So, the sum of nine and x =9+x

What is a number?

A number is a mathematical object used to count, measure, and label.

Six times the sum of nine and a number means multiplication of 6 and the number 9+x.

Six times the sum of nine and the number x=6(9+x)

Six times the sum of nine and  the number x =54+6x

Hence, Six times the sum of nine and the number x is 54+6x.

To get more about numbers visit:

https://brainly.com/question/21855

Ratio of three angles of a triangle is 1 : 2 : 3. Find the angles.

Answers

Answer:

30⁰ , 60⁰ , 90⁰

Step-by-step explanation:

Let the angles x , 2x , 3x

By angle sum property:

x + 2x + 3x = 180⁰

6x = 180⁰

x = 30⁰

Angles are : 30⁰ , 60⁰ , 90⁰

Let one angle be x, the second 2x and the third 3x.

x + 2x + 3x = 180 degrees (angle sum property of a triangle)

6x = 180 degrees

x = 180/6

x = 30 degrees

So one angle is 30°, the second is 60°, and the third is 90°.

Please help me any one can help me please I need help please help me

Answers

Answer:

537.84

Step-by-step explanation:

First find the area of the box(covered with pizza): 30*30=900

Next, find the area of the circle: 24/2= radius; 12^2*3.13=452.16

Now we must find the area of the box that isn't covered in pizza.

To do this we can subtract the area of the box(covered in pizza) and the area of the pizza: 900-452.16

=537.84

During the summer, jody earns 10$ per hour babysitting and 15$ per hour doing yardwork. This week she worked 34 hours and earned 410$. If x represents the number of hours she babysat and y represents the number of hours she did yardwork, which system of equations models this situation?

Answers

Answer:

x + y = 34

10*x + 15*y = 410

Step-by-step explanation:

The equation system that models the situation would be made up of two equations:

The first equation would be the total number of hours for both jobs, that is, the number of babysat work hours and the number of yardwork work hours equals 34 total hours.

x + y = 34

The second equation would be the total amount of money earned, in this case it would be to multiply the earnings of each job by the number of hours of each job and add it up and you should give a total of 410 which are your total earnings.

10 * x + 15 * y = 410

Therefore the system of equations would be:

x + y = 34

10 * x + 15 * y = 410

Other Questions
POSSIBLE POINTS: 1In the US, the average citizen receives an annual dose of 360 mrem of radiation. If eating a banana creates a 0.01 mrem dose, how many bananasare equivalent to the annual dose of an average citizen?720 bananas36.000 bananas25,000 bananas180,000 bananas What type of probability is used in this scenario a bag has 10 red marbles 7 blue marbles and 12 yellow marbles the probability of drawing a yellow at random is 12/29 Ellie goes out to dinner with her family. The total bill is $57. They decide to leave a 20% tip. How much money should they leave as their tip? How did the Great Migration impact the lives of women and African American throughout WWI? Please answer this question as a fraction, will mark you the brainliest if ur right! The person who drew this cartoon was a supporter ofBritish imperialism. What message was he trying toconvey?Uprisings in the Balkans must be contained, or thegreat powers could be threatened.Nationalism must continue, or the great powerscould be threatened.Militarism must continue, or the great powers couldbe threatened.Uprisings in Europe must be contained, or theBalkans could be threatened. Question 2 of 105 PointsWhen the theory that the Sun goes around Earth was replaced with the theorythat Earth goes around the Sun, this was an example of aAnswer hereSOBY Which of the following correctly describes the Filter procedure for managing spreadsheet data? Check all that apply.Filter allows you to view only the information you want to see.Filter deletes rows of data you do not need in your spreadsheet.Filter allows you to filter text and numeric data.Filter temporarily hides rows of data that you do not need to view. Graph a line with a slope of -5 that contains thepoint (-3,-4).y6..2-6-424-6 Which statement about basaltic magma is false? a. It forms when rocks in the upper mantle meltb. It contains small amounts of silica c. It has high viscosity d. It forms on oceanic and continental crust Which details could be included in a biography of Frida Kahlo? Check all that apply.1.a description of the first self-portrait Kahlo painted2.a story about Kahlos childhood pet3.an account of the accident in which Kahlo was seriously injured4.Kahlos impressions of other artists of the time period5.a description of a cousin Kahlo never met6.an example of how Kahlo influenced artists who came after her With the wire back at its initial location, you connect a second identical battery in series with the first one. When you close the switch, how does the new angle of deflection of the north pole of the compass needle compare to its initial deflection what is not a healthy way to express yourself How can quotations support a structured analysis of a character? 3710Which expression has a negative value?O 2+12(-3)(-8)10-(-18)-35-5Wakandi telur Network economics Select one: A. applies the law of diminishing returns to communities of users. B. balances the high cost of adding new members to a community against the lower cost of using network infrastructure. C. sees the cost of adding new members as inconsequential. D. applies traditional economics to networked usersh The highest and lowest scores on a test taken by 80 students are within 19 points of the average of the exam. The average for this exam was a 72. Set up an equation to solve for the highest and lowest scores. Show your original equation and the process used to find the highest and lowest exam score. How many oxide ions are in 0.55 grams of titanium oxide? Choose the answer to complete each statementThe slope of the line isThe y-intercept is atThe graph represents the function(3, 4)(0, -3).. Identify the direct object highlighted in the passage.